Difference between revisions of "2023 SSMO Team Round Problems/Problem 1"
(→Solution) |
|||
(2 intermediate revisions by one other user not shown) | |||
Line 2: | Line 2: | ||
Let <math>(a, b, c, d)</math> be a permutation of <math>(2, 0, 2, 3)</math>. Find the largest possible value of <math>a^b + b^c + c^d + d^a</math> | Let <math>(a, b, c, d)</math> be a permutation of <math>(2, 0, 2, 3)</math>. Find the largest possible value of <math>a^b + b^c + c^d + d^a</math> | ||
− | ==Solution== | + | ==Solution 1== |
+ | WLOG, assume that <math>a = 0</math>. Therefore, we have <math>(a,b,c,d) = (0,2,2,3),(0,2,3,2),</math> or <math>(0,3,2,2).</math> The value of <math>a^b+b^c+c^d+d^a</math> for these three permutations are <math>13,18,</math> and <math>14,</math> respectively, meaning the greatest possible sum is <math>\boxed{18}.</math> | ||
− | We can assume because of the symmetry that <math>a=0</math>. Then, the problem is reduced to <math>1+b^c+c^d</math>. Since there are only <math>3</math> possible permutations for <math>b</math>, <math>c</math>, and <math>d</math>, we can try them and find that the maximum possible value is obtained when <math>b=2</math>, <math>c=3</math>, and <math>d=2</math>. Therefore, the answer is <math>1+2^3+3^2=\boxed{18}</math> | + | ~SMO_Team |
+ | |||
+ | ==Solution 2== | ||
+ | |||
+ | We can assume because of the symmetry that <math>a=0</math>. Then, the problem is reduced to <math>1+b^c+c^d</math>. Since there are only <math>3</math> possible permutations for <math>b</math>, <math>c</math>, and <math>d</math>, we can try them and find that the maximum possible value is obtained when <math>b=2</math>, <math>c=3</math>, and <math>d=2</math>. Therefore, the answer is <math>1+2^3+3^2=\boxed{18}</math>. | ||
+ | |||
+ | ~alexanderruan |
Latest revision as of 21:14, 9 September 2025
Problem
Let be a permutation of
. Find the largest possible value of
Solution 1
WLOG, assume that . Therefore, we have
or
The value of
for these three permutations are
and
respectively, meaning the greatest possible sum is
~SMO_Team
Solution 2
We can assume because of the symmetry that . Then, the problem is reduced to
. Since there are only
possible permutations for
,
, and
, we can try them and find that the maximum possible value is obtained when
,
, and
. Therefore, the answer is
.
~alexanderruan